Question about continuity postulates

In summary: So if you require a physical motivation for the continuity of \psi, just say that if it were discontinuous, the derivative could be infinite, and since the derivative is the momentum, this is unphysical.In summary, the constraint that the wavefunction must be continuous and smooth is simply an axiom of the theory, but it can also be justified physically by considering the consequences of a discontinuous wavefunction. This condition allows for enough equations to determine the coefficients of the wavefunction and can be demonstrated experimentally through the probability current being continuous.
  • #1
thesm
3
0
I had a search for an answer but I turned up nothing, if this has been covered before could someone point me in the right direction?

To the question.

I'm studying QM at the moment but I'm having trouble with two of the postulates. Is the constraint that the wavefunction must be continuous and smooth (continuous first derivatives) based on anything more, or is it simply an axiom of the theory? To me it seems like the condition was placed because it means you then have enough equations to determine the coefficients of the wavefunction.

If it is one of the axioms of the theory I'd be interested to know of any experimental ways of demonstrating that this is the case.

Thanks
Ryan

(long time lurker, first time poster)
 
Physics news on Phys.org
  • #2
"If it is one of the axioms of the theory I'd be interested to know of any experimental ways of demonstrating that this is the case."

tunnel effect
 
  • #3
Ok, so it's an axiom, thanks. I got the impression from my lecturer that the continutity conditions should make sense physically, like that it should be obvious that a wavefunction is smooth and continuous, but it wasn't making any sense like that to me.

And of course, quantum tunneling, thanks again.
 
  • #4
I am not a mathemetician, but if I remember correctly a second order differential equation has to has continuous solutions along with a continuous 1st derivative.
 
  • #5
Ok, I feel stupid now.
I was going to ask why should this apply on the boundary, since if the wave function was undefined outside this then the continuous/smooth condition couldn't be added, but one of the postulates is that the wave function is defined everywhere.
Thanks for the responses.
 
  • #6
A more abstract way of looking at it is to say that the operators are what's really important, and continuously differentiable functions are a good class of things upon which they can act.
 
  • #7
thesm said:
I had a search for an answer but I turned up nothing, if this has been covered before could someone point me in the right direction?
To the question.
I'm studying QM at the moment but I'm having trouble with two of the postulates. Is the constraint that the wavefunction must be continuous and smooth (continuous first derivatives) based on anything more, or is it simply an axiom of the theory? To me it seems like the condition was placed because it means you then have enough equations to determine the coefficients of the wavefunction.
If it is one of the axioms of the theory I'd be interested to know of any experimental ways of demonstrating that this is the case.
Thanks
Ryan
(long time lurker, first time poster)

Well, first thing's first:

the wavefunction has nothing to do with the postulates of (nonrelativistic) QM.

Then, once u pick the irreducible representation of the Born-Jordan CCR-s to be the "wave mechanics" formalism, then the Hilbert space for one particle becomes [itex] L^{2}\left(\mathbb{R}^{3} \right) [/itex]. That's where the physics takes place. Since the free particle hamiltonian is an unbounded, densly defined, self-adjoint linear operator which, up to a constant is

[tex] -\nabla^{2} [/itex]

, then the normalization condition and the differential form of the common operators (linear momentum, angular momentum, hamiltonian) impose certain restrictions upon the element of the Hilbert space. [itex] C^{2} [/itex] class on a certain domain of [itex] \mathbb{R}^{3} [/itex] is a good requirement on the wavefunction. And of course, being a Schwartz functional is another one.

A good survey on this matter is found in Reed & Simon.

Daniel.
 
  • #8
Probability current will be continuous,

if the gradient of the wave function has a smooth derivative.

Would that not be a good reason?


(see http://electron6.phys.utk.edu/qm1/modules/m4/probability.htm for the maths)
 
Last edited by a moderator:
  • #9
Just for completeness sake, it may be interesting to know that if the potential term in the time-independent Schrodinger equation is a Dirac delta function, the solution will have a "derivative jump", i.e. the first derivative will have a finite discontinuity, even when the wavefunction itself remains continuous.

Zz.
 
  • #10
It seems to me obvious that the wavefunction should be continuous and differentiable. To say it is discontinuous at a certain point in space would be physically meaningless, since we have no measurable access to phenomena in an infinitely small region as a 'point', which has no dimensions. We can't measure such sudden change in such a function over an interval of, say, [itex]10^{-50}[/itex]m.
Any function which varies significantly over a very small interval will give the same results as a discontinuous one.
For example, a square well potential is physically impossible, but it will do to describe a potential which varies (continuously and smoothly) over a distance which is short compared to the wavelength of the particle.

If you want 'mathematical proof' of this, you can do as Griffiths does and integrate the (time-independent) Schrödinger equation over a small interval:

[tex]-\frac{\hbar^2}{2m}\int_{-\epsilon}^{+\epsilon}\frac{d^2\psi}{dx^2}dx+\int_{-\epsilon}^{+\epsilon}V(x)\psi(x)dx=E\int_{-\epsilon}^{+\epsilon}\psi(x)dx[/tex]
The first integral is [itex]d\psi / dx}[/itex] evaluated at the two endpoints. The last integral is zero in the limit [itex]\epsilon \to 0[/itex], since [itex]\psi[/itex] is bounded and the width of the 'window' goes to zero. So:
[tex]\lim_{\epsilon \to 0} \Delta \left(\frac{d\psi}{dx}\right)=\frac{2m}{\hbar^2}\lim_{\epsilon \to 0}\int_{-\epsilon}^{+\epsilon}V(x)\psi(x)dx[/tex]

V(x) is normally (and in any physical case) finite and thus [itex]d\psi / dx[/itex] is continuous. If V(x) is infinite however (as in the case of a delta function potential), the derivative of [itex]\psi[/itex] can undergo a discontinuity.
 
Last edited:

1. What are continuity postulates?

Continuity postulates are fundamental principles in mathematics and physics that describe the behavior of continuous functions and systems. They state that a function or system is continuous if it has no abrupt changes or discontinuities.

2. How many continuity postulates are there?

There are three main continuity postulates: the limit postulate, the intermediate value postulate, and the extreme value postulate. These postulates are used to define and prove the continuity of functions and systems.

3. Why are continuity postulates important?

Continuity postulates are important because they allow us to study and understand the behavior of continuous functions and systems. They provide a framework for solving problems and making predictions in various fields, such as mathematics, physics, and engineering.

4. How do continuity postulates relate to calculus?

Continuity postulates are closely related to the concept of limits in calculus. The limit postulate states that a function is continuous if the limit of the function exists at a given point. This is a key concept in calculus, as it allows us to calculate rates of change and solve problems involving continuous functions.

5. Are there any exceptions to continuity postulates?

There are certain functions and systems that do not follow the continuity postulates, such as discontinuous functions and chaotic systems. However, these exceptions are often studied and analyzed separately in order to understand their unique behaviors and properties.

Similar threads

  • Special and General Relativity
2
Replies
41
Views
2K
  • Quantum Physics
4
Replies
130
Views
8K
  • Quantum Physics
Replies
13
Views
992
  • Topology and Analysis
Replies
5
Views
2K
  • Quantum Physics
Replies
24
Views
1K
Replies
49
Views
3K
  • Special and General Relativity
Replies
3
Views
472
  • Quantum Interpretations and Foundations
Replies
28
Views
2K
  • Quantum Physics
Replies
29
Views
4K
Replies
35
Views
4K
Back
Top